Respuesta :

Answer:

Option (1)

Step-by-step explanation:

In a geometric series,

[tex]\sum_{n=0}^{\infty}ar^n=a+ar^1+ar^2+....[/tex]

Here r = [tex]\frac{a_n}{a_{n-1}}[/tex]

For |r| < 1, series will converge.

For |r| > 1, series will diverge.

Option (1)

Given geometric series is,

[tex]\frac{1}{2}+\frac{1}{4}+ \frac{1}{8}+ \frac{1}{16}.......[/tex]

Common ratio = [tex]\frac{\frac{1}{4}}{\frac{1}{2} }[/tex]

                        = [tex]\frac{1}{2}[/tex]

Since, [tex]\frac{1}{2}<1[/tex]

Series will converge.

Option (2)

[tex]\frac{1}{2}+1+2+4+......[/tex]

r = [tex]\frac{1}{\frac{1}{2}}=2[/tex]

Since, 2 > 1,

Series will diverge.

Option (3)

[tex]\frac{1}{2}+ \frac{3}{2}+ \frac{9}{2}+ \frac{27}{2}+......[/tex]

Common ratio 'r' = [tex]\frac{\frac{3}{2} }{\frac{1}{2} }=3[/tex]

Since, 3 > 1,

Series will diverge.

Option (4)

[tex]\frac{1}{2}+3+18+108+.....[/tex]

Common ratio 'r' = [tex]\frac{3}{\frac{1}{2}}=6[/tex]

Since, 6 > 1

Series will diverge.

Therefore, Option (1) will be the correct option.

Answer:

A

Step-by-step explanation: